Bonjour,

je rencontre une difficulté à prouver la solution indiquée dans cette question :

Capture d’écran 2018-12-10 à 4.46.14 PM.jpg

Capture d’écran 2018-12-10 à 4.47.14 PM.png

Si je calcule l'égalité vectorielle sur les 3 composantes, je trouve :







Comme c'est une solution stationnaire selon le vecteur e_y, il faudrait que je trouve une relation du type mais je cale.

Pensez-vous que pour le gradient de pression, le fait de le considérer uniquement selon la composante z (Oz) est la bonne hypothèse ?

Merci

ps : le contexte est de la physique stellaire mais j'ai posté ici car ça semble avant tout un problème de mécanique des fluides.